www.vorkurse.de
Ein Projekt von vorhilfe.de
Die Online-Kurse der Vorhilfe

E-Learning leicht gemacht.
Hallo Gast!einloggen | registrieren ]
Startseite · Mitglieder · Teams · Forum · Wissen · Kurse · Impressum
Forenbaum
^ Forenbaum
Status Mathe-Vorkurse
  Status Organisatorisches
  Status Schule
    Status Wiederholung Algebra
    Status Einführung Analysis
    Status Einführung Analytisc
    Status VK 21: Mathematik 6.
    Status VK 37: Kurvendiskussionen
    Status VK Abivorbereitungen
  Status Universität
    Status Lerngruppe LinAlg
    Status VK 13 Analysis I FH
    Status Algebra 2006
    Status VK 22: Algebra 2007
    Status GruMiHH 06
    Status VK 58: Algebra 1
    Status VK 59: Lineare Algebra
    Status VK 60: Analysis
    Status Wahrscheinlichkeitst

Gezeigt werden alle Foren bis zur Tiefe 2

Navigation
 Startseite...
 Neuerdings beta neu
 Forum...
 vorwissen...
 vorkurse...
 Werkzeuge...
 Nachhilfevermittlung beta...
 Online-Spiele beta
 Suchen
 Verein...
 Impressum
Das Projekt
Server und Internetanbindung werden durch Spenden finanziert.
Organisiert wird das Projekt von unserem Koordinatorenteam.
Hunderte Mitglieder helfen ehrenamtlich in unseren moderierten Foren.
Anbieter der Seite ist der gemeinnützige Verein "Vorhilfe.de e.V.".
Partnerseiten
Weitere Fächer:

Open Source FunktionenplotterFunkyPlot: Kostenloser und quelloffener Funktionenplotter für Linux und andere Betriebssysteme
Forum "Uni-Stochastik" - Konfidenzintervalle
Konfidenzintervalle < Stochastik < Hochschule < Mathe < Vorhilfe
Ansicht: [ geschachtelt ] | ^ Forum "Uni-Stochastik"  | ^^ Alle Foren  | ^ Forenbaum  | Materialien

Konfidenzintervalle: Frage (beantwortet)
Status: (Frage) beantwortet Status 
Datum: 19:51 So 15.01.2012
Autor: MattiJo

Aufgabe
Sei [mm] (X_1, [/mm] ..., [mm] X_n) [/mm] eine Zufallsstichprobe. Bestimmen Sie

a) ein asymptotisches Konfidenzintervall zum Niveau [mm] \gamma \in [/mm] [0, 1] für den Parameter [mm] \Theta, [/mm] falls [mm] X_i \in [/mm] U(0, [mm] \Theta), [/mm] i = 1, ..., n, [mm] \Theta [/mm] > 0,

b) ein asymptotisches Konfidenzintervall zum Niveau [mm] \gamma \in [/mm] [0, 1] für den Parameter [mm] \Theta, [/mm] falls [mm] X_i \in Exp(\Theta), [/mm] i = 1, ..., n, [mm] \Theta [/mm] > 0,

c) Zeigen Sie, dass die Statistik [mm] T(X_1, [/mm] ..., [mm] X_n) [/mm] = [mm] 2n\Theta \overline X_n [/mm] einer [mm] \chi^2_{2n} [/mm] - Verteilung genügt und bestimmen Sie damit ein exaktes Konfidenzintervall zum Niveau [mm] \gamma \in [/mm] [0, 1] für den Parameter [mm] \Theta, [/mm] falls [mm] X_i \in Exp(\Theta), [/mm] i = 1, ..., n, [mm] \Theta [/mm] > 0

Hallo zusammen,

einmal mehr bräuchte ich einen Gedankenanstoß für die Aufgabe a).
Als Ansatz habe ich []hier die Formeln unmittelbar unter Formel (64) für [mm] \overline \Theta [/mm] und [mm] \underline \Theta [/mm] .... jedoch bin ich mir bei den einzugebenden Parametern nicht sicher! n ist schließlich undefiniert und nicht klar, welches Quantil...

        
Bezug
Konfidenzintervalle: Antwort
Status: (Antwort) fertig Status 
Datum: 21:14 So 15.01.2012
Autor: luis52

Moin,

das Beispiel, auf das du dich beziehst, behandelt den Fall der Poisson-Verteilung.  Fuer die Verteilung in Teil a) lautet der Ansatz

[mm] $\lim_{n\to\infty}P\left(-z_{1-\alpha/2}\le\sqrt{n}\dfrac{\bar X-\operatorname{E}[X]}{\sqrt{\operatorname{Var}[X]}}\le z_{1-\alpha/2}\right)=\lim_{n\to\infty} P\left(-z_{1-\alpha/2}\le\sqrt{n}\dfrac{\bar X-\theta/2}{\sqrt{12\theta^2}}\le z_{1-\alpha/2}\right)=1-\alpha$. [/mm]

vg Luis


Bezug
                
Bezug
Konfidenzintervalle: Frage (beantwortet)
Status: (Frage) beantwortet Status 
Datum: 22:31 So 15.01.2012
Autor: MattiJo

Vielen Dank soweit!

Wie komme ich zum 0,975-Quantil der Gleichverteilung?
Muss ich selbst die Quantilfunktion aufstellen? Bei sämtlichen Verteilungen kann ich die Quantile aus Tabellen entnehmen...

Bezug
                        
Bezug
Konfidenzintervalle: Antwort
Status: (Antwort) fertig Status 
Datum: 22:41 So 15.01.2012
Autor: luis52


> Vielen Dank soweit!
>  
> Wie komme ich zum 0,975-Quantil der Gleichverteilung?
>  Muss ich selbst die Quantilfunktion aufstellen? Bei
> sämtlichen Verteilungen kann ich die Quantile aus Tabellen
> entnehmen...

[mm] $z_{1-\alpha/2}$ [/mm] ist das [mm] $(1-\alpha)\cdot100$%-Quantil [/mm] der *Standardnormalverteilung*...

vg Luis

Bezug
                
Bezug
Konfidenzintervalle: Frage (beantwortet)
Status: (Frage) beantwortet Status 
Datum: 00:20 Mo 16.01.2012
Autor: MattiJo

Vielen Dank soweit!

Der Ansatz leuchtet mir ein - unser Intervall soll so bestimmt werden, dass [mm] \Theta [/mm] mit einer Wahrscheinlichkeit von [mm] \gamma [/mm] = 1 - [mm] \alpha [/mm] eben drin ist.
Soll ich nun die im Argument enthaltene Ungleichung nach [mm] \Theta [/mm] auflösen, oder wie komm ich zum Ziel?

Bezug
                        
Bezug
Konfidenzintervalle: Antwort
Status: (Antwort) fertig Status 
Datum: 08:58 Mo 16.01.2012
Autor: luis52


>  Soll ich nun die im Argument enthaltene Ungleichung nach
> [mm]\Theta[/mm] auflösen,

Ja.

> oder wie komm ich zum Ziel?

vg Luis




Bezug
                                
Bezug
Konfidenzintervalle: Frage (beantwortet)
Status: (Frage) beantwortet Status 
Datum: 13:23 Mo 16.01.2012
Autor: MattiJo

Ungleichungen sind nicht meine Stärke - ich komme als Lösung auf

[mm] \bruch{\wurzel{n}\cdot \overline{X}}{\bruch{\wurzel{n}}{2} + \wurzel{12}z_{1-\bruch{\alpha}{2}}} \le \Theta \le \bruch{\wurzel{n}\cdot \overline{X}}{\bruch{\wurzel{n}}{2} - \wurzel{12}z_{1-\bruch{\alpha}{2}}} [/mm]

Klingt das plausibel?

Dann mach ich mich mal nach dem selben Schema an die (b)....
bzw. hierzu gleich mal die Frage,

stimmt hier dann der Ansatz

[mm] \limes_{n\rightarrow\infty} P(-z_{1-\bruch{\alpha}{2}}\le \wurzel{n} \bruch{\overline{X} - \bruch{1}{\Theta}}{\wurzel{\bruch{1}{\Theta^2}}} \le z_{1-\bruch{\alpha}{2}}) [/mm] = $1- [mm] \alpha$ [/mm]

?

Bezug
                                        
Bezug
Konfidenzintervalle: Antwort
Status: (Antwort) fertig Status 
Datum: 15:19 Mo 16.01.2012
Autor: luis52


> Ungleichungen sind nicht meine Stärke - ich komme als
> Lösung auf
>  
> [mm]\bruch{\wurzel{n}\cdot \overline{X}}{\bruch{\wurzel{n}}{2} + \wurzel{12}z_{1-\bruch{\alpha}{2}}} \le \Theta \le \bruch{\wurzel{n}\cdot \overline{X}}{\bruch{\wurzel{n}}{2} - \wurzel{12}z_{1-\bruch{\alpha}{2}}}[/mm]Eingabefehler: "\left" und "\right" müssen immer paarweise auftreten, es wurde aber ein Teil ohne Entsprechung gefunden (siehe rote Markierung)


>
> Klingt das plausibel?

Kann sein. Mathematica liefert die Grenzen $\frac{2 n \bar x\mp 8    \sqrt{3} \sqrt{n} \bar x z}{n-48    z^2}\right\}$ (ohne Gewaehr).


>
> Dann mach ich mich mal nach dem selben Schema an die
> (b)....
>  bzw. hierzu gleich mal die Frage,
>  
> stimmt hier dann der Ansatz
>  
> [mm]\limes_{n\rightarrow\infty} P(-z_{1-\bruch{\alpha}{2}}\le \wurzel{n} \bruch{\overline{X} - \bruch{1}{\Theta}}{\wurzel{\bruch{1}{\Theta^2}}} \le z_{1-\bruch{\alpha}{2}})[/mm]
> = [mm]1- \alpha[/mm]


[ok] Es wird ja! ;-)

vg Luis



Bezug
                                                
Bezug
Konfidenzintervalle: Frage (beantwortet)
Status: (Frage) beantwortet Status 
Datum: 15:22 Mo 16.01.2012
Autor: MattiJo

Super danke!! :-)

Kannst du evtl. mir noch den Ansatz zur c) liefern - dann bin ich glücklich ;-)

Bezug
                                                        
Bezug
Konfidenzintervalle: Antwort
Status: (Antwort) fertig Status 
Datum: 20:01 Mo 16.01.2012
Autor: luis52


> Kannst du evtl. mir noch den Ansatz zur c) liefern - dann
> bin ich glücklich ;-)

Nutze aus, dass [mm] $n\bar [/mm] X$ eine []Erlang-Verteilung besitzt.

vg Luis


Bezug
Ansicht: [ geschachtelt ] | ^ Forum "Uni-Stochastik"  | ^^ Alle Foren  | ^ Forenbaum  | Materialien


^ Seitenanfang ^
www.vorkurse.de
[ Startseite | Mitglieder | Teams | Forum | Wissen | Kurse | Impressum ]